2006 SMT/General Problems/Problem 16

Revision as of 18:19, 14 January 2020 by Dividend (talk | contribs) (Created page with "==Solution== The condition we are looking for is <math>35(n-1)\equiv 0 \mod 360</math>, because if any other <math>A_n = A_1</math> than <math>35(n-1)</math> must be a multip...")
(diff) ← Older revision | Latest revision (diff) | Newer revision → (diff)

Solution

The condition we are looking for is $35(n-1)\equiv 0 \mod 360$, because if any other $A_n = A_1$ than $35(n-1)$ must be a multiple of 360 as we have rotated $(n-1)$ by the time we place $A_n$.

\[35(n-1)\equiv 0\mod 360 \Rightarrow 5(n-1)\equiv 0\mod 360 \Rightarrow x\equiv 73,1\mod 360\]

We can ignore the case of $n=1$ because be are looking for an $n\neq1$. Therefore, our answer is the second lowest option, $n=73$ or $\boxed{A_{73}}$